Final Exam Practice Questions

Pataasin ang iyong marka sa homework at exams ngayon gamit ang Quizwiz!

Which stage of the grief process outlined by Elizabeth Kübler-Ross is characterized by confusion and a decreased ability to function independently? 1.Denial 2.Anger 3.Bargaining 4.Acceptance

2.Anger

Which is the most likely characteristic of the disequilibrium stage in the grief process hypothesized by John Bowlby? 1.Feeling of shock 2.Preoccupation with the loss 3.Establishment of new patterns of organization 4.Somatic complaints

2.Preoccupation with the loss

14. Which individual is most likely to be below the poverty level in the United States? 1. A 70-year-old Hispanic woman living alone 2. A 72-year-old African American man living alone 3. A 68-year-old Asian American woman living with family 4. A 75-year-old Latino American man living with family

1. A 70-year-old Hispanic woman living alone

Which behavioral symptom is most likely observed in a client with neurocognitive disorder (NCD)? 1. Delirium 2. Decubiti 3. Paranoia 4. Amnesia

1. Delirium

12. An older client has met the criteria for a diagnosis of major depressive disorder. The client does not respond to antidepressant medications. Which therapeutic intervention should a nurse anticipate will be ordered for this client? 1. Electroconvulsive therapy (ECT) 2. Neuroleptic therapy 3. An antiparkinsonian agent 4. An anxiolytic agent

1. Electroconvulsive therapy (ECT)

Which neurotransmitter is oversecreted in a client with neurocognitive disorder (NCD), leading to neuronal degeneration and cell death? 1. Glutamate 2. Serotonin 3. Dopamine 4. Acetylcholine

1. Glutamate

16. According to Reichard, Livson, and Peterson, a client is classified as an "armored man." Which personality description led to this classification? 1. Rigid and stable, presenting a strong silent front 2. Passive-dependent individuals who lean on others for support 3. Aggressiveness is common, as is suspicion of others 4. Animosity is turned inward on themselves

1. Rigid and stable, presenting a strong silent front

After assessing the cognitive capacity of a client with Alzheimer's disease (AD), the nurse concludes that the client is in the fourth stage of the disease. Which finding supports the nurse's conclusion? 1. The client is unable to understand current news events. 2. The client is unable to recall address and phone numbers. 3. The client is unable to recall names of family members. 4. The client is unable to plan or organize office work.

1. The client is unable to understand current news events

While caring for a cognitively impaired client, the nurse provides a cane and instructs the client about its use. Which outcome will the nurse expect from this intervention? 1. The client will not experience physical injury. 2. The client might be able to show wandering behavior. 3. The client will maintain a therapeutic relationship with the nurse. 4. The client might be able to perform the activities of daily living independently.

1. The client will not experience physical injury

Which is the social worker's role in the hospice care setting? 1.Assisting the client's family with financial issues 2.Providing directions to the client regarding pain management 3.Providing companionship 4.Providing bereavement counseling to family members of the deceased

1.Assisting the client's family with financial issues

Which type of grief response does a bereaved individual most likely experience if the individual is strongly influenced by his or her culture to maintain a "stiff upper lip" in response to the loss? 1.Delayed grief 2.Distorted grief 3.Prolonged grief 4.Anticipatory grief

1.Delayed grief

Which is the major difference between normal grieving and a maladaptive grieving response? 1.Feeling worthless or having low self-esteem 2.Feeling considerable anger toward the environment 3.Feeling helpless and frustrated 4.Taking years to adapt to a new way of life

1.Feeling worthless or having low self-esteem

Which is the most commonly available type of hospice care in the United States? 1.Home-based care 2.Freestanding institutions that provide both inpatient and home care 3.Hospital-affiliated institutional care 4.Nursing home-affiliated institutional care

1.Home-based care

Which is an example of delayed grief? 1.Overreaction to another person's loss 2.Detachment from a dying family member even before the death has occurred 3.Depressive mood disorder as a reaction to loss 4.Extreme feelings of powerlessness

1.Overreaction to another person's loss

Which behavior is exhibited by a bereaved individual in stage II of the grief process outlined by George Engel? 1.Regressing to a state of helplessness 2.Feeling numb 3.Participating in the funeral 4.Imitating admired qualities of the lost entity

1.Regressing to a state of helplessness

In which stage of grief outlined by Elizabeth Kübler-Ross does a client acknowledge the loss but hold out hope for additional alternatives? 1.Stage III 2.Stage V 3.Stage II 4.Stage I

1.Stage III

Which do most preschoolers and kindergartners believe regarding the concept of death? 1.They believe death is reversible. 2.They may believe that death is contagious. 3.They are interested in the physical aspects of dying and the final disposition of the body. 4.They generally have an attitude of immortality.

1.They believe death is reversible.

6. A student nurse asks the instructor, "Which psychiatric disorder is most likely initially diagnosed in the elderly?" Which instructor response gives the student accurate information? 1. "Schizophrenia is most likely diagnosed later in life." 2. "Major depressive disorder is most likely diagnosed later in life." 3. "Phobic disorder is most likely diagnosed later in life." 4. "Dependent personality disorder is most likely diagnosed later in life."

2. "Major depressive disorder is most likely diagnosed later in life."

4. A nursing instructor is teaching about reminiscence therapy. What student statement indicates that learning has occurred? 1. "Reminiscence therapy is a group in which participants create collages representing significant aspects of their lives." 2. "Reminiscence therapy encourages members to share both positive and negative significant life memories to promote resolution." 3. "Reminiscence therapy is a social group where members chat about past events and future plans." 4. "Reminiscence therapy encourages members to share positive memories of significant life transitions."

2. "Reminiscence therapy encourages members to share both positive and negative significant life memories to promote resolution."

Which action does the nurse implement to prevent the risk of accidental trauma while caring for a client with neurocognitive disorder (NCD)? 1. Keeping all lights off at night 2. Positioning the bed as low as possible 3. Using clocks and calendars with large numbers 4. Arranging the client's room away from the nursing station to avoid disturbance

2. Positioning the bed as low as possible

A bereaved individual feels such intense sadness at the passing of the lost entity that the individual is unable to properly manage daily living activities. The individual also experiences chronic somatic symptoms. Which sort of grief response does this individual experience? 1.Normal grief 2.Distorted grief 3.Inhibited grief 4.Anticipatory grief

2.Distorted grief

13. A nurse is charting assessment information about a 70-year-old client. According to the U.S. Census Bureau, what term would the nurse use to describe this client? 1. The nurse should document using the term older. 2. The nurse should document using the term elderly. 3. The nurse should document using the term aged. 4. The nurse should document using the term very old.

2. The nurse should document using the term elderly.

Which medication is used in the treatment of agitation? 1. Donepezil 2. Risperidone 3. Galantamine 4. Rivastigmine

2. Risperidone

9. An older, emaciated client is brought to an emergency department by the client's caregiver. The client has bruises and abrasions on shoulders and back in multiple stages of healing. When directly asked about these symptoms, which type of client response should a nurse anticipate? 1. The client will honestly reveal the nature of the injuries. 2. The client may deny or minimize the injuries. 3. The client may have forgotten what caused the injuries. 4. The client will ask to be placed in a nursing home.

2. The client may deny or minimize the injuries.

While caring for a client diagnosed with neurocognitive disorder (NCD), the nurse remains calm, undemanding, and avoids forcing the client to perform activities that he or she is refusing. Which outcome will the nurse observe in the client after this intervention? 1. The client stays calm without wandering. 2. The client remains calm with minimally agitated behavior. 3. The client will experience fewer episodes of acute confusion. 4. The client will maintain reality orientation to the best of his or her cognitive ability.

2. The client remains calm with minimally agitated behavior

Children of which age group are most likely to believe that their thoughts or behaviors caused a person to become sick or to die? 1.2 years of age 2.3 to 5 years of age 3.6 to 9 years of age 4.10 to 12 years of age

2.3 to 5 years of age

1. A client has recently been placed in a long-term care facility, because of marked confusion and inability to perform most activities of daily living (ADLs). Which nursing intervention is most appropriate to maintain the client's self-esteem? 1. Leave the client alone in the bathroom to test ability to perform self-care. 2. Assign a variety of caregivers to increase potential for socialization. 3. Allow client to choose between two different outfits when dressing for the day. 4. Modify the daily schedule often to maintain variety and decrease boredom.

3. Allow client to choose between two different outfits when dressing for the day.

11. An older client is exhibiting symptoms of major depressive disorder. A physician is considering prescribing an antidepressant. Which physiological problem should make a nurse question this medication regime? 1. Altered cortical and intellectual functioning 2. Altered respiratory and gastrointestinal functioning 3. Altered liver and kidney functioning 4. Altered endocrine and immune system functioning

3. Altered liver and kidney functioning

10. A client in the middle stage of Alzheimer's disease has difficulty communicating because of cognitive deterioration. Which nursing intervention is appropriate to improve communication? 1. Discourage attempts at verbal communication owing to increased client frustration. 2. Increase the volume of the nurse's communication responses. 3. Verbalize the nurse's perception of the implied communication. 4. Encourage the client to communicate by writing.

3. Verbalize the nurse's perception of the implied communication.

How long does the acute phase of normal grieving last in a 40-year-old adult? 1.2 to 3 weeks 2.4 to 5 weeks 3.6 to 8 weeks 4.12 to 14 weeks

3.6 to 8 weeks

A client who is a heavy smoker is informed by the health-care provider that his or her lung cancer has metastasized and he or she has only a few months to live. The individual says, "Please God, if I am spared this time, I will never touch a cigarette again." Which stage of the grief response is the client experiencing, based on the stages of grief outlined by Elizabeth Kübler-Ross? 1.Depression 2.Acceptance 3.Bargaining 4.Denial

3.Bargaining

Which behavior is most likely to be observed in the restitution stage of the grief process as outlined by George Engel? 1.Refusal to acknowledge the loss 2.Regression to a state of helplessness 3.Participation in rituals associated with loss 4.Preoccupation with the loss

3.Participation in rituals associated with loss

5. A couple both reside in a long-term care facility. The husband is admitted to the psychiatric unit after physically abusing his wife. He states, "My wife is having an affair with a young man, and I want it investigated." Which is the appropriate nursing response? 1. "Your wife is not having an affair. What makes you think that?" 2. "Why do you think that your wife is having an affair?" 3. "Your wife has told us that these thoughts have no basis in fact." 4. "I understand that you are upset. We will talk about it."

4. "I understand that you are upset. We will talk about it."

A client has neurocognitive disorder (NCD). The client's wife asks the nurse, "My husband is a chronic smoker. Can I give him cigarettes and a lighter?" How should the nurse respond to the client's partner? 1. "Yes, you can give the cigarettes and lighter to your husband." 2. "Please tell your husband to smoke only when there is no one around." 3. "We are not supposed to encourage smoking in clients with NCD." 4. "I will keep the cigarettes at my work station and will dispense them to the client when appropriate."

4. "I will keep the cigarettes at my work station and will dispense them to the client when appropriate."

15. According to Reichard, Livson, and Peterson, which classification of the personalities of older men describe passive-dependent individuals who are content to lean on others for support, to disengage, and to let most of life's activities pass them by? 1. "Mature men" personalities 2. "Armored men" personalities 3. "Self-haters" personalities 4. "Rocking chair" personalities

4. "Rocking chair" personalities

Which client would be highly sensitive to extrapyramidal effects of antipsychotics? 1. A client with vascular NCD 2. A client with NCD due to head injury 3. A client with frontotemporal NCD 4. A client with Lewy Body NCD

4. A client with Lewy Body NCD

A client is admitted into the psychiatric unit with the complaint of pressured and incoherent speech. On further interaction, the nurse finds that the client is suffering from delirium. What is the priority nursing intervention in this situation? 1. Maintain a low level of stimuli. 2. Monitor the behavior of the client. 3. Provide reorientation and assurance to the client. 4. Determine and correct the underlying causes.

4. Determine and correct the underlying causes.

Which intervention should the nurse implement while caring for a client with neurocognitive disorder (NCD) and diagnosed with wandering behavior? 1. Storing frequently used items out of the client's reach 2. Teaching the client to hold onto a hand-railing 3. Maintaining a low level of stimuli in the environment 4. Keeping the client on a structured schedule of recreational activities

4. Keeping the client on a structured schedule of recreational activities

Which stage of Alzheimer's disease (AD) is characterized by sundowning? 1. Forgetfulness 2. Mild cognitive decline 3. Severe cognitive decline 4. Moderate-to-severe cognitive decline

4. Moderate-to-severe cognitive decline

Which diagnostic test is performed to evaluate the progression of Alzheimer's disease (AD) in a client? 1. Electroencephalogram (EEG) 2. Magnetic resonance imaging (MRI) 3. Computed tomography (CT) scanning 4. Positron emission tomography (PET) scanning

4. PET scan

A client with Alzheimer's disease (AD) has impaired speech and communication. The client's chest radiograph shows the presence of inflamed alveoli. What does the nurse infer from these findings? 1. The client is in the fourth stage of the disease. 2. The client is in the fifth stage of the disease. 3. The client is in the sixth stage of the disease. 4. The client is in the seventh stage of the disease.

4. The client is in the seventh stage of the disease

The nurse is caring for a client with self-care deficits. Which outcome will demonstrate the effectiveness of the nursing intervention? 1. The client will interpret the environment accurately. 2. The client will be able to make his or her needs known to caregivers. 3. The client will use the measures provided to maintain reality orientation. 4. The client will participate in the activities of daily life with the assistance of caregivers.

4. The client will participate in the activities of daily life with the assistance of caregivers

For a client with Alzheimer's disease, what is the condition called when he or she creates events to fill in memory gaps? 1. Delusion 2. Contusion 3. Hallucination 4. Confabulation

4. confabulation

Children of which age are most likely to personify death in their minds in the form of a monster? 1.10-year-olds 2.2-year-olds 3.4-year-olds 4.7-year-olds

4.7-year-olds

The nurse observes that a terminally ill client is becoming increasingly quiet and withdrawn. Therapeutic intervention concludes that the client is not depressed. Which stage of grief response is the client experiencing, as outlined by Elizabeth Kübler-Ross' stages of grief response? 1.Denial 2.Anger 3.Bargaining 4.Acceptance

4.Acceptance

Which condition is a predisposing factor in the development of depressive disorder in older adults? 1.Prolonged grieving 2.Inhibited grief 3.Distorted grieving 4.Cumulative grief

4.Cumulative grief

Which child would be most difficult to diagnose for a neurodevelopmental disorder? A. 3 yr old B. 5 yr old C. 8 yr old D. 12 yr old

A. 3 yr old

A child diagnosed with autism spectrum disorder has the nursing diagnosis of disturbed personal identity. Which outcome would best address this client's diagnosis? A. The client will name own body parts as separate from others by day 5. B. The client will establish a means of communicating personal needs by discharge. C. The client will initiate social interactions with caregivers by day 4. D. The client will not harm self or others by discharge.

A. The client will name own body parts as separate from others by day 5. An appropriate outcome for this client is to name own body parts as separate from others. The nurse should assist the client in the recognition of separateness during self-care activities such as dressing and feeding. The long-term goal for disturbed personal identity is for the client to develop an ego identity. KEY: Cognitive Level: Application | Integrated Processes: Nursing Process: Planning | Client Need: Psychosocial Integrity

A woman returns home after delivering a stillborn infant to find that neighbors have dismantled the nursery that she and her husband planned. According to Worden, how should a nurse expect the neighbors action to affect the womans grieving task completion? A. This action may hamper the woman from accepting the reality of the loss. B. This action would help the woman forget the sorrow and move on with life. C. This action communicates full support from her neighbors. D. This action would motivate the woman to look to the future and not the past.

A. This action may hamper the woman from accepting the reality of the loss.

2. A woman describes a history of physical and emotional abuse in intimate relationships. Which additional factor should a nurse suspect? A. The woman may be exhibiting a controlled response pattern. B. The woman may have a history of childhood neglect. C. The woman may be exhibiting codependent characteristics. D. The woman might be a victim of incest.

D. The woman might be a victim of incest. The nurse should suspect that this client might be a victim of incest. Women in abusive relationships often grew up in abusive homes. KEY: Cognitive Level: Application | Integrated Processes: Nursing Process: Assessment | Client Need: Psychosocial Integrity

Clara, an 80 year old woman, says to the nurse, "I'm all alone now. my husband is gone. my best friend is gone. my daughter is busy with her work and family. I might as well just go, too." Which is the best response by the nurse? a. "are you thinking you want to die, Clara?" b. "You have lots to live for, Clara." c. "Cheer up, Clara. You have so much to be thankful for." d. "Tell me about your family, Clara."

a. "are you thinking you want to die, Clara?"

A school nurse notices bruises and scars on a child's body. The nurse suspects that the child is being physically abused. How should the nurse proceed with this information? a. As a health-care worker, report the suspicion to the Department of Health and Human Services. b. Check Jana again in a week and see if there are any new bruises. c. Meet with Jana's parents and ask them how Jana got the bruises. d. Initiate paperwork to have Jana placed in foster care.

a. As a health-care worker, report the suspicion to the Department of Health and Human Services.

Sharon, a woman with multiple cuts and abrasions, arrives at the emergency department (ED) with her three small children. She tells the nurse her husband inflicted these wounds on her. She says, "I didn't want to come. I'm really okay. He only does this when he has too much to drink. I just shouldn't have yelled at him." The best response by the nurse is, a. "How often does he drink too much?" b. "It is not your fault. You did the right thing by coming here." c. "How many times has he done this to you?" d. "He is not a good husband. You have to leave him before he kills you."

b. "It is not your fault. You did the right thing by coming here."

A woman who was sexually assaulted six months ago by a man with whom she was acquainted has since been attending a support group for survivors of rape. From this group, she has learned that the most likely reason the man raped her was that a. because he had been drinking, he was not in control of his actions. b. he had not had sexual relations with a girl in many months. c. he was predisposed to become a rapist by virtue of the poverty conditions under which he was reared. d. he was expressing power and dominance by means of sexual aggression and violence

d. he was expressing power and dominance by means of sexual aggression and violence

What is the most appropriate way to communicate with an elderly person who is deaf in his right ear? a. Speak loudly into his left ear b. Speak to him from a position on his left side c. Speak face-to-face in a high-pitched voice d. speak face-to-face in a low-pitched voice

d. speak face-to-face in a low-pitched voice

A school nurse notices bruises and scars on a child's body, but the child refuses to say how she received them. Another way in which the nurse can get information from the child is to a. have her evaluated by the school psychologist. b. tell her she may select a "treat" from the treat box (e.g., sucker, balloon, junk jewelry) if she answers the nurse's questions. c. explain to her that if she answers the questions, she may stay in the nurse's office and not have to go back to class. d. use a "family" of dolls to role-play the child's family with her.

d. use a "family" of dolls to role-play the child's family with her.

A soldier who has been missing in action for more than 3 years finally returns home to his wife. She had believed him to be dead and had grieved over his loss. Now he is upset due to the realization that she had started to move on with her life. Which type of grief response has the wife exhibited in this situation? 1.Anticipatory grief 2.Inhibited grief 3.Distorted grief 4.Chronic grief

1.Anticipatory grief

2. A son, who recently brought his extremely confused parent to a nursing home for admission, reports feelings of guilt. Which is the appropriate nursing response? 1. "Support groups are held here on Mondays for children of residents in similar situations." 2. "You did what you had to do. I wouldn't feel guilty if I were you." 3. "Support groups are available to low-income families." 4. "Your parent is doing just fine. We'll take very good care of him."

1. "Support groups are held here on Mondays for children of residents in similar situations."

Which nursing intervention is beneficial for a client who has disturbed thought processes? 1. Initiating reminiscence therapy 2. Keeping a dim light in the client's room at night 3. Keeping the bed as low as possible 4. Speaking loudly and clearly by looking directly into the client's eyes

1. Initiating reminiscence therapy

Which behavioral sign does the nurse find in a client diagnosed with mild neurocognitive disorder (NCD)? 1. Modest cognitive decline in learning and memory 2. Ability to carry out motor activities 3. Disorientation to surroundings 4. Inability to perform everyday activities independently

1. Modest cognitive decline in learning and memory

In which situation would the grief response be most difficult for the bereaved individual? 1. The individual has experienced a number of recent losses. 2. The individual knew beforehand that the loss was going to occur. 3. The individual lost an elderly person who was close to him or her. 4. The individual participated in the rituals associated with loss within his or her culture.

1. The individual has experienced a number of recent losses.

While caring for a client with impaired cognitive and psychomotor functioning, the nurse frequently orients the client to place, time, and situation. What is the rationale behind this intervention? 1. To ensure safety of the client 2. To provide orientation to reality 3. To minimize confusion in the client 4. To support the client in performing independent actions

1. To ensure safety of the client

Family members of a client who is dying begin to pull away from and provide less support to the client who is dying. As a result, the client who is dying expresses feelings of loneliness and isolation. Which is the most likely problematic grief response exhibited by the family? 1.Anticipatory grief response 2.Inhibited grief response 3.Chronic grief response 4.Exaggerated grief response

1.Anticipatory grief response

Which type of grief response acts as a defense for some individuals to ease the burden of loss when it actually occurs? 1.Anticipatory grief response 2.Delayed grief response 3.Exaggerated grief response 4.Normal grief response Rationales

1.Anticipatory grief response

Which is a sign of clinical depression in a bereaved individual? 1.Experiencing transient physical symptoms 2.Not expressing anger directly 3.Expressing feelings of guilt over some aspect of the loss 4.Experiencing a mixture of "good and bad days"

2.Not expressing anger directly

3. A family asks why their father is attending activity groups at the long-term care facility. The son states, "My father worked hard all of his life. He just needs some rest at this point." Which is the appropriate nursing response? 1. "I'm glad we discussed this. We'll excuse him from the activity groups." 2. "The groups benefit your father by providing social interaction, sensory stimulation, and reality orientation." 3. "The groups are optional. Only clients at high functioning levels would benefit." 4. "If your father doesn't go to these activity groups, he will be at high risk for developing cognitive problems."

2. "The groups benefit your father by providing social interaction, sensory stimulation, and reality orientation."

7. An older client attending an adult day care program suddenly begins reporting dizziness, weakness, and confusion. What should be the initial nursing intervention? 1. Implement complete bedrest. 2. Advocate for a complete physical exam. 3. Address self-esteem needs. 4. Advocate for individual psychotherapy.

2. Advocate for a complete physical exam.

Which stage of the grief process outlined by Elizabeth Kübler-Ross is characterized by elevated anxiety levels and an idealized image of what has been lost? 1.Denial 2.Anger 3.Bargaining 4.Acceptance

2.Anger

Which is a normal grief response? 1.Inhibited grieving 2.Anticipatory grieving 3.Exaggerated grieving 4.Prolonged grieving

2.Anticipatory grieving

A client has recently gone through a mastectomy as a part of her treatment for breast cancer. She requires therapeutic intervention while she works through her feelings of grief regarding the mastectomy. Which stage of the grief response outlined by Elizabeth Kübler-Ross is the client most likely experiencing? 1.Acceptance 2.Depression 3.Bargaining 4.Anger

2.Depression

In which stage of the grief response outlined by George Engel does an individual feel anger toward the people in the environment whom the individual holds accountable for the loss? 1.Shock and disbelief 2.Developing awareness 3.Restitution 4.Resolution of the loss

2.Developing awareness

Which statement is true regarding the concept of death held by most children between the ages of 6 and 9 years? 1.They believe death is reversible. 2.They may believe that death is contagious. 3.They are interested in the physical aspects of dying and the final disposition of the body. 4.They generally have an attitude of immortality.

2.They may believe that death is contagious.

A client is in the late stage of Alzheimers disease. To address the clients symptoms, which nursing intervention should take priority? 1. Improve cognitive status by encouraging involvement in social activities. 2. Decrease social isolation by providing group therapies. 3. Promote dignity by providing comfort, safety, and self-care measures. 4. Facilitate communication by providing assistive devices.

3. Promote dignity by providing comfort, safety, and self-care

Which statement made by the client's husband makes the nurse suspect the client is in the mild cognitive decline stage of Alzheimer's disease (AD)? 1. "My wife forgot our child's birthday." 2. "My wife is unable to recognize family members." 3. "My wife has difficulty recalling my name." 4. "My wife is unable to perform the daily activities of life."

3. "My wife has difficulty recalling my name."

8. An older client who lives with a caregiver is admitted to an emergency department with a fractured arm. The client is soaked in urine and has dried fecal matter on lower extremities. The client is 6 feet tall and weighs 120 pounds. Which condition should the nurse suspect? 1. Inability for the client to meet self-care needs 2. Alzheimer's disease 3. Abuse and/or neglect 4. Caregiver role strain

3. Abuse and/or neglect

Which manifestation indicates neurocognitive disorder (NCD) due to prion disease? 1. Displacement of the brain with onset of seizures and amnesia 2. Loss of nerve cells in substantia nigra with tremors in upper extremities 3. Rapid progression of the disease accompanying movement disturbances and ataxia 4. Damage of basal ganglia and cerebral cortex with myoclonus, depression, and apathy

3. Rapid progression of the disease accompanying movement disturbances and ataxia

How does it help a grieving client if the nurse encourages the client to reach out to familiar religious practices for support? 1.It increases the client's sense of self-worth. 2.It helps the client trust the nurse-client relationship. 3.It puts life back into a realistic perspective. 4.It helps the client find comfort in religious rituals with which he or she is familiar.

4.It helps the client find comfort in religious rituals with which he or she is familiar.

In which stage of the grief response outlined by George Engel does the bereaved individual idealize the concept of the loss? 1.Stage V 2.Stage II 3.Stage III 4.Stage IV

4.Stage IV

In which situation is the grief response facilitated? 1.The individual was strongly dependent on the lost entity. 2.The individual had a love-hate relationship with the lost entity. 3.The individual's physical health was unstable at the time of the loss. 4.The individual knew that the loss was imminent.

4.The individual knew that the loss was imminent.

A geriatric nurse is teaching student nurses about the risk factors for development of delirium in older adults. Which student statement indicates that learning has occurred? A. "Taking multiple medications may lead to adverse interactions or toxicity." B. "Age-related cognitive changes may lead to alterations in mental status." C. "Lack of rigorous exercise may lead to decreased cerebral blood flow." D. "Decreased social interaction may lead to profound isolation and psychosis."

A. "Taking multiple medications may lead to adverse interactions or toxicity." The nurse should identify that taking multiple medications may lead to adverse reactions or toxicity and put an older adult at risk for the development of delirium. Symptoms of delirium include difficulty sustaining and shifting attention. The client with delirium is disoriented to time and place and may also have impaired memory. KEY: Cognitive Level: Application | Integrated Processes: Nursing Process: Evaluation | Client Need: Physiological Integrity

A nursing instructor presents a case study in which a 3-year-old child is in constant motion and is unable to sit still during story time. The instructor asks a student to evaluate this child's behavior. Which student response indicates an appropriate evaluation of the situation? A. "This child's behavior must be evaluated according to developmental norms." B. "This child has symptoms of attention deficit hyperactivity disorder." C. "This child has symptoms of the early stages of autistic disorder." D. "This child's behavior indicates possible symptoms of oppositional defiant disorder."

A. "This child's behavior must be evaluated according to developmental norms." The student's evaluation of the situation is appropriate when indicating a need for the client to be evaluated according to developmental norms. Guidelines for determining whether emotional problems exist in a child should consider if the behavioral manifestations are not age-appropriate, deviate from cultural norms, or create deficits or impairments in adaptive functioning. KEY: Cognitive Level: Application | Integrated Processes: Nursing Process: Evaluation | Client Need: Psychosocial Integrity

A nursing instructor is teaching about donepezil (Aricept). A student asks, "How does this work? Will this cure Alzheimer's disease (AD)?" Which is the appropriate instructor reply? A. "This medication delays the destruction of acetylcholine, a chemical in the brain necessary for memory processes. Although most effective in the early stages, it serves to delay, but not stop, the progression of the AD." B. "This medication encourages production of acetylcholine, a chemical in the brain necessary for memory processes. It delays the progression of the disease." C. "This medication delays the destruction of dopamine, a chemical in the brain necessary for memory processes. Although most effective in the early stages, it serves to delay, but not stop, the progression of the AD." D. "This medication encourages production of dopamine, a chemical in the brain necessary for memory processes. It delays the progression of the disease."

A. "This medication delays the destruction of acetylcholine, a chemical in the brain necessary for memory processes. Although most effective in the early stages, it serves to delay, but not stop, the progression of the AD." The most appropriate response by the instructor is to explain that donepezil (Aricept) delays the destruction of acetylcholine, a chemical in the brain necessary for memory processes. Although most effective in the early stages, it serves to delay, but not stop, the progression of AD. KEY: Cognitive Level: Analysis | Integrated Processes: Teaching/Learning | Client Need: Physiological Integrity: Pharmacological and Parenteral Therapies

When planning care for a client, which medication classification should a nurse recognize as effective in the treatment of Tourette's disorder? A. Antipsychotic medications B. Antimanic medications C. Tricyclic antidepressant medications D. Monoamine oxidase inhibitor medications

A. Antipsychotic medications The nurse should recognize that antipsychotic medications are effective in the treatment of Tourette's disorder. These medications are used to reduce the severity of tics and are most effective when combined with psychosocial therapy. Risperidone (Risperdal) has been shown to reduce symptoms by 21% to 61%. KEY: Cognitive Level: Application | Integrated Processes: Nursing Process: Planning | Client Need: Physiological Integrity: Pharmacological and Parenteral Therapies

A client diagnosed with neurocognitive disorder exhibits progressive memory loss, diminished cognitive functioning, and verbal aggression upon experiencing frustration. Which nursing intervention is most appropriate? A. Schedule structured daily routines. B. Minimize environmental lighting. C. Organize a group activity to present reality. D. Explain the consequences for aggressive behaviors.

A. Schedule structured daily routines. The most appropriate nursing intervention for this client is to schedule structured daily routines. A structured routine will reduce frustration and thereby reduce verbal aggression. KEY: Cognitive Level: Application | Integrated Processes: Nursing Process: Implementation | Client Need: Psychosocial Integrity

An adolescent client who was diagnosed with conduct disorder at the age of 8 is sentenced to juvenile detention after bringing a gun to school. How should the nurse apply knowledge of conduct disorder to this client's situation? A. Childhood-onset conduct disorder is more severe than the adolescent-onset type, and these individuals likely develop antisocial personality disorder in adulthood. B. Childhood-onset conduct disorder is caused by a difficult temperament, and the child is likely to outgrow these behaviors by adulthood. C. Childhood-onset conduct disorder is diagnosed only when behaviors emerge before the age of 5, and therefore improvement is likely. D. Childhood-onset conduct disorder has no treatment or cure, and children diagnosed with this disorder are likely to develop progressive oppositional defiant disorder.

A. Childhood-onset conduct disorder is more severe than the adolescent-onset type, and these individuals likely develop antisocial personality disorder in adulthood. The nurse should apply knowledge of conduct disorder to determine that childhood-onset conduct disorder is more severe than adolescent-onset type. These individuals are likely to develop antisocial personality disorder in adulthood. Individuals with this subtype are usually boys and frequently display physical aggression and have disturbed peer relationships. KEY: Cognitive Level: Application | Integrated Processes: Nursing Process: Assessment | Client Need: Psychosocial Integrity

What term should a nurse use when describing a response to grieving that includes a sudden physical collapse and paralysis, and which cultural group would be associated with this behavior? A. Falling out in the African American culture B. Body rocking in the Vietnamese American culture C. Conversion disorder in the Jewish American culture D. Spirit possession in the Native American culture

A. Falling out in the African American culture

8. A college student was sexually assaulted when out on a date. After several weeks of crisis intervention therapy, which client statement should indicate to a nurse that the student is handling this situation in a healthy manner? A. I know that it was not my fault. B. My boyfriend has trouble controlling his sexual urges. C. If I dont put myself in a dating situation, I wont be at risk. D. Next time I will think twice about wearing a sexy dress.

A. I know that it was not my fault. The client who realizes that sexual assault was not her fault is handling the situation in a healthy manner. The nurse should provide nonjudgmental listening and communicate statements that instill trust and validate self-worth. KEY: Cognitive Level: Application | Integrated Processes: Nursing Process: Evaluation | Client Need: Psychosocial Integrity

A nursing instructor is teaching about the typical grieving behaviors of Chinese Americans. Which student statement would indicate that more instruction is needed? A. In this culture, the color red is associated with death and is considered bad luck. B. In this culture, there is an innate fear of death. C. In this culture, emotions are not expressed openly. D. In this culture, death and bereavement are centered on ancestor worship.

A. In this culture, the color red is associated with death and is considered bad luck.

3. Which statement made by an emergency department nurse indicates accurate knowledge of domestic violence? A. Power and control are central to the dynamic of domestic violence. B. Poor communication and social isolation are central to the dynamic of domestic violence. C. Erratic relationships and vulnerability are central to the dynamic of domestic violence. D. Emotional injury and learned helplessness are central to the dynamic of domestic violence.

A. Power and control are central to the dynamic of domestic violence. The nurse accurately states that power and control are central to the dynamic of domestic violence. Battering is defined as a pattern of coercive control founded on physical and/or sexual violence or threat of violence. The typical abuser is very possessive and perceives the victim as a possession. KEY: Cognitive Level: Application | Integrated Processes: Assessment | Client Need: Psychosocial Integrity

13. Which assessment data should a school nurse recognize as signs of physical neglect? A. The child is often absent from school and seems apathetic and tired. B. The child is very insecure and has poor self-esteem. C. The child has multiple bruises on various body parts. D. The child has sophisticated knowledge of sexual behaviors.

A. The child is often absent from school and seems apathetic and tired. The nurse should recognize that a child who is often absent from school and seems apathetic and tired might be a victim of neglect. Other indicators of neglect are stealing food or money, lacking medical or dental care, being consistently dirty, lacking sufficient clothing, or stating that there is no one home to provide care. KEY: Cognitive Level: Application | Integrated Processes: Nursing Process: Assessment | Client Need: Safe and Effective Care Environment

1. A kindergarten student is frequently violent toward other children. A school nurse notices bruises and burns on the childs face and arms. What other symptom should indicate to the nurse that the child might have been physically abused? A. The child shrinks at the approach of adults. B. The child begs or steals food or money. C. The child is frequently absent from school. D. The child is delayed in physical and emotional development.

A. The child shrinks at the approach of adults. The nurse should determine that a child who shrinks at the approach of adults in addition to having bruises and burns might be a victim of abuse. Whether or not the adult intended to harm the child, maltreatment should be considered. KEY: Cognitive Level: Application | Integrated Processes: Nursing Process: Assessment | Client Need: Safe and Effective Care Environment

A child diagnosed with autism spectrum disorder has the nursing diagnosis of disturbed personal identity. Which outcome would best address this clients diagnosis? A. The client will name own body parts as separate from others by day 5. B. The client will establish a means of communicating personal needs by discharge. C. The client will initiate social interactions with caregivers by day 4. D. The client will not harm self or others by discharge.

A. The client will name own body parts as separate from others by day 5

After an adolescent diagnosed with attention deficit-hyperactivity disorder (ADHD) begins methylphenidate (Ritalin) therapy, a nurse notes that the adolescent loses 10 pounds in a 2-month period. What is the best explanation for this weight loss? A. The pharmacological action of Ritalin causes a decrease in appetite. B. Hyperactivity seen in ADHD causes increased caloric expenditure. C. Side effects of Ritalin cause nausea; therefore, caloric intake is decreased. D. Increased ability to concentrate allows the client to focus on activities rather than food.

A. The pharmacological action of Ritalin causes a decrease in appetite. The pharmacological action of Ritalin causes a decrease in appetite that often leads to weight loss. Methylphenidate (Ritalin) is a central nervous symptom stimulant that serves to increase attention span, control hyperactive behaviors, and improve learning ability for clients diagnosed with ADHD. KEY: Cognitive Level: Application | Integrated Processes: Nursing Process: Evaluation | Client Need: Physiological Integrity: Pharmacological and Parenteral Therapies

A nursing instructor presents a case study in which a 3-year-old child is in constant motion and is unable to sit still during story time. The instructor asks a student to evaluate this childs behavior. Which student response indicates an appropriate evaluation of the situation? A. This childs behavior must be evaluated according to developmental norms. B. This child has symptoms of attention deficit hyperactivity disorder. C. This child has symptoms of the early stages of autistic disorder. D. This childs behavior indicates possible symptoms of oppositional defiant disorder.

A. This child's behavior must be evaluated according to developmental norms

After an adolescent diagnosed with attention deficit-hyperactivity disorder (ADHD) begins methylphenidate (Ritalin) therapy, a nurse notes that the adolescent loses 10 pounds in a 2-month period. What is the best explanation for this weight loss? A. The pharmacological action of Ritalin causes a decrease in appetite. B. Hyperactivity seen in ADHD causes increased caloric expenditure. C. Side effects of Ritalin cause nausea; therefore, caloric intake is decreased. D. Increased ability to concentrate allows the client to focus on activities rather than food.

A. the pharmacological action of Ritalin causes a decrease in appetite

A 6-year-old client is prescribed methylphenidate (Ritalin) for a diagnosis of attention deficit-hyperactivity disorder (ADHD). When teaching the parents about this medication, which nursing statement explains how Ritalin works? A. "Ritalin's sedation side effect assists children by decreasing their energy level." B. "How Ritalin works is unknown. Although it is a stimulant, it does combat the symptoms of ADHD." C. "Ritalin helps the child focus by decreasing the amount of dopamine in the basal ganglia and neuron synapse." D. "Ritalin decreases hyperactivity by increasing serotonin levels."

B. "How Ritalin works is unknown. Although it is a stimulant, it does combat the symptoms of ADHD." It is unknown how Ritalin works, but even though it is a stimulant, it does decrease hyperactivity in individuals diagnosed with ADHD. KEY: Cognitive Level: Application | Integrated Processes: Teaching/Learning | Client Need: Physiological Integrity: Pharmacological and Parenteral Therapies

A child has been diagnosed with autism spectrum disorder. The distraught mother cries out, "I'm such a terrible mother. What did I do to cause this?" Which nursing reply is most appropriate? A. "Researchers really don't know what causes autistic disorder, but the relationship between autistic disorder and fetal alcohol syndrome is being explored." B. "Poor parenting doesn't cause autism. Research has shown that abnormalities in brain structure and/or function are to blame. This is beyond your control." C. "Research has shown that the mother appears to play a greater role in the development of this disorder than the father." D. "Lack of early infant bonding with the mother has shown to be a cause of autistic disorder. Did you breastfeed or bottle-feed?"

B. "Poor parenting doesn't cause autism. Research has shown that abnormalities in brain structure and/or function are to blame. This is beyond your control." The most appropriate reply by the nurse is to explain to the parent that autism spectrum disorder is believed to be caused by abnormalities in brain structure and/or function, not poor parenting. Autism spectrum disorder occurs in approximately 6 per 1,000 children and is about four times more likely to occur in boys. KEY: Cognitive Level: Application | Integrated Processes: Teaching/Learning | Client Need: Psychosocial Integrity

A nurse is caring for a client who has early stage Alzheimer's disease and a new prescription for donepezil. The nurse should include which of the following statements when teaching the client about the medication? A. "You should avoid taking over-the-counter acetaminophen while on donepezil." B. "You should take this medication before going to bed at the end of the day." C. "You will be screened for underlying kidney disease prior to starting donepezil." D. "You should stop taking donepezil if you experience nausea or diarrhea."

B. "You should take this medication before going to bed at the end of the day."

A nurse has taken report for the evening shift on an adolescent inpatient unit. Which client should the nurse address first? A. A client diagnosed with oppositional defiant disorder being sexually inappropriate with staff B. A client diagnosed with conduct disorder who is verbally abusing a peer in the milieu C. A client diagnosed with conduct disorder who is demanding special attention from staff D. A client diagnosed with attention deficit disorder who has a history of self-mutilation

B. A client diagnosed with conduct disorder who is verbally abusing a peer in the milieu A client diagnosed with conduct disorder who is verbally abusing a peer in the milieu presents a potential safety concern that would need to be addressed by the nurse immediately. KEY: Cognitive Level: Application | Integrated Processes: Nursing Process: Implementation | Client Need: Safe and Effective Care Environment: Management of Care

A physician orders methylphenidate (Ritalin) for a child diagnosed with attention deficit-hyperactivity disorder (ADHD). Which information about this medication should the nurse provide to the parents? A. If one dose of Ritalin is missed, double the next dose. B. Administer Ritalin to the child after breakfast. C. Administer Ritalin to the child just prior to bedtime. D. A side effect of Ritalin is decreased ability to learn.

B. Administer Ritalin to the child after breakfast

A physician orders methylphenidate (Ritalin) for a child diagnosed with attention deficit-hyperactivity disorder (ADHD). Which information about this medication should the nurse provide to the parents? A. If one dose of Ritalin is missed, double the next dose. B. Administer Ritalin to the child after breakfast. C. Administer Ritalin to the child just prior to bedtime. D. A side effect of Ritalin is decreased ability to learn.

B. Administer Ritalin to the child after breakfast. The nurse should instruct the parents to administer Ritalin to the child after breakfast. Ritalin is a central nervous system stimulant and can cause decreased appetite. Central nervous system stimulants can also temporarily interrupt growth and development. KEY: Cognitive Level: Application | Integrated Processes: Nursing Process: Implementation | Client Need: Physiological Integrity: Pharmacological and Parenteral Therapies

A client has an IQ of 47. Which nursing diagnosis best addresses a client problem associated with this degree of intellectual disability? A. Risk for injury R/T self-mutilation B. Altered social interaction R/T nonadherence to social convention C. Altered verbal communication R/T delusional thinking D. Social isolation R/T severely decreased gross motor skills

B. Altered social interaction R/T nonadherence to social convention The appropriate nursing diagnosis associated with this degree of intellectual disability is altered social interaction R/T nonadherence to social convention. A client with an IQ of 47 would be diagnosed with moderate intellectual disability and may also experience some limitations in speech communications. KEY: Cognitive Level: Analysis | Integrated Processes: Nursing Process: Analysis | Client Need: Psychosocial Integrity

A child has been recently diagnosed with mild intellectual disability (ID). What information about this diagnosis should the nurse include when teaching the child's mother? A. Children with mild ID need constant supervision. B. Children with mild ID develop academic skills up to a sixth-grade level. C. Children with mild ID appear different from their peers. D. Children with mild ID have significant sensory-motor impairment.

B. Children with mild ID develop academic skills up to a sixth-grade level. The nurse should inform the child's mother that children with mild ID develop academic skills up to a sixth-grade level. Individuals with mild ID are capable of independent living, capable of developing social skills, and have normal psychomotor skills. KEY: Cognitive Level: Application | Integrated Processes: Teaching/Learning | Client Need: Health Promotion and Maintenance

A 6-year-old client is prescribed methylphenidate (Ritalin) for a diagnosis of attention deficit-hyperactivity disorder (ADHD). When teaching the parents about this medication, which nursing statement explains how Ritalin works? A. Ritalins sedation side effect assists children by decreasing their energy level. B. How Ritalin works is unknown. Although it is a stimulant, it does combat the symptoms of ADHD. C. Ritalin helps the child focus by decreasing the amount of dopamine in the basal ganglia and neuron synapse. D. Ritalin decreases hyperactivity by increasing serotonin levels.

B. How Ritalin works is unknown. Although it is a stimulant, it does combat the symptoms of ADHD.

Which nursing intervention should be prioritized when caring for a child diagnosed with intellectual disability? A. Encourage the parents to always prioritize the needs of the child. B. Modify the child's environment to promote independence and encourage impulse control. C. Delay extensive diagnostic studies until the child is developmentally mature. D. Provide one-on-one tutorial education in a private setting to decrease overstimulation.

B. Modify the child's environment to promote independence and encourage impulse control. The nurse should prioritize modifying the child's environment to promote independence and encourage impulse control. This intervention is related to the nursing diagnosis self-care deficit. Positive reinforcement can serve to increase self-esteem and encourage repetition of behaviors. KEY: Cognitive Level: Analysis | Integrated Processes: Nursing Process: Implementation | Client Need: Safe and Effective Care Environment

Which statement demonstrates that a parent understands the diagnosis of ADHD? A. My child will never be able to graduate or go to college but may be able to learn a vocational skill B. My child's performance will improve in a structured setting that provides rewards for appropriate behavior C. Nothing is wrong with my child. The school hasn't provided qualified teachers and classroom settings D. My child is just going through a stage. This problem will go away with time

B. My child's performance will improve in a structured setting that provides rewards for appropriate behavior

Which symptom should a nurse identify that would differentiate clients diagnosed with neurocognitive disorders from clients diagnosed with amnesic disorders? A. Neurocognitive disorders involve disorientation that develops suddenly, whereas amnestic disorders develop more slowly. B. Neurocognitive disorders involve impairment of abstract thinking and judgment, whereas amnestic disorders do not. C. Neurocognitive disorders include the symptom of confabulation, whereas amnestic disorders do not. D. Both neurocognitive disorders and profound amnesia typically share the symptom of disorientation to place, time, and self.

B. Neurocognitive disorders involve impairment of abstract thinking and judgment, whereas amnestic disorders do not. Neurocognitive disorders involve impairment of abstract thinking and judgment. Amnestic disorders are characterized by an inability to learn new information and to recall previously learned information, with no impairment in higher cortical functioning or personality change. KEY: Cognitive Level: Analysis | Integrated Processes: Nursing Process: Assessment | Client Need: Physiological Integrity

Which grieving behaviors should a nurse anticipate when caring for a Southwest Navajo Indian client? A. Celebrating the life of a deceased person with festivities and revelry B. Not expressing grief openly and reluctance to touch a dead body C. Holding a prayerful vigil for a week following the persons death D. Expressing grief openly and publicly and erecting an altar in the home to honor the dead

B. Not expressing grief openly and reluctance to touch a dead body

A child has been diagnosed with autism spectrum disorder. The distraught mother cries out, Im such a terrible mother. What did I do to cause this? Which nursing reply is most appropriate? A. Researchers really dont know what causes autistic disorder, but the relationship between autistic disorder and fetal alcohol syndrome is being explored. B. Poor parenting doesnt cause autism. Research has shown that abnormalities in brain structure and/or function are to blame. This is beyond your control. C. Research has shown that the mother appears to play a greater role in the development of this disorder than the father. D. Lack of early infant bonding with the mother has shown to be a cause of autistic disorder. Did you breastfeed or bottle-feed?

B. Poor parenting doesnt cause autism. Research has shown that abnormalities in brain structure and/or function are to blame. This is beyond your control.

14. An anorexic client states to a nurse, My father has recently moved back to town. Since that time the client has experienced insomnia, nightmares, and panic attacks that occur nightly. She has never married or dated and lives alone. What should the nurse suspect? A. Possible major depressive disorder B. Possible history of childhood incest C. Possible histrionic personality disorder D. Possible history of childhood bulimia

B. Possible history of childhood incest The nurse should suspect that this client might have a history of childhood incest. Adult survivors of incest are at risk for developing posttraumatic stress disorder, sexual dysfunction, somatization disorders, compulsive sexual behavior disorders, depression, anxiety, eating disorders, and substance abuse disorders. KEY: Cognitive Level: Application | Integrated Processes: Nursing Process: Assessment | Client Need: Psychosocial Integrity

A preschool child is admitted to a psychiatric unit with a diagnosis of autism spectrum disorder. To help the child feel more secure on the unit, which intervention should a nurse include in this clients plan of care? A. Encourage and reward peer contact. B. Provide consistent caregivers. C. Provide a variety of safe daily activities. D. Maintain close physical contact throughout the day.

B. Provide consistent caregivers

A preschool child is admitted to a psychiatric unit with a diagnosis of autism spectrum disorder. To help the child feel more secure on the unit, which intervention should a nurse include in this client's plan of care? A. Encourage and reward peer contact. B. Provide consistent caregivers. C. Provide a variety of safe daily activities. D. Maintain close physical contact throughout the day.

B. Provide consistent caregivers. The nurse should provide consistent caregivers as part of the plan of care for a child diagnosed with autism spectrum disorder. Children diagnosed with autism spectrum disorder have an inability to trust. Providing consistent caregivers allows the client to develop trust and a sense of security. KEY: Cognitive Level: Application | Integrated Processes: Nursing Process: Implementation | Client Need: Psychosocial Integrity

Which nursing intervention related to self-care would be most appropriate for a teenager diagnosed with moderate intellectual disability? A. Meeting all of the client's self-care needs to avoid injury B. Providing simple directions and praising client's independent self-care efforts C. Avoiding interference with the client's self-care efforts in order to promote autonomy D. Encouraging family to meet the client's self-care needs to promote bonding

B. Providing simple directions and praising client's independent self-care efforts Providing simple directions and praise is an appropriate intervention for a teenager diagnosed with moderate intellectual disability. Individuals with moderate intellectual disability can perform some activities independently and may be capable of academic skill to a second-grade level. KEY: Cognitive Level: Application | Integrated Processes: Nursing Process: Implementation | Client Need: Safe and Effective Care Environment

Which behavioral approach should a nurse utilize when caring for children diagnosed with disruptive behavior disorders? A. Involving parents in designing and implementing the treatment process B. Reinforcing positive actions to encourage repetition of desired behaviors C. Providing opportunities to learn appropriate peer interactions D. Administering psychotropic medications to improve quality of life

B. Reinforcing positive actions to encourage repetition of desired behaviors

Which behavioral approach should a nurse utilize when caring for children diagnosed with disruptive behavior disorders? A. Involving parents in designing and implementing the treatment process B. Reinforcing positive actions to encourage repetition of desired behaviors C. Providing opportunities to learn appropriate peer interactions D. Administering psychotropic medications to improve quality of life

B. Reinforcing positive actions to encourage repetition of desired behaviors The nurse should reinforce positive actions to encourage repetition of desired behaviors when caring for children diagnosed with a disruptive behavior disorder. Behavior therapy is based on the concepts of classical conditioning and operant conditioning. KEY: Cognitive Level: Application | Integrated Processes: Nursing Process: Implementation | Client Need: Psychosocial Integrity

4. A client is brought to an emergency department after being violently raped. Which nursing action is appropriate? A. Discourage the client from discussing the event, as this may lead to further emotional trauma. B. Remain nonjudgmental and actively listen to the clients description of the event. C. Meet the clients self-care needs by assisting with showering and perineal care. D. Provide cues, based on police information, to encourage further description of the event.

B. Remain nonjudgmental and actively listen to the clients description of the event. The most appropriate nursing action is to remain nonjudgmental and actively listen to the clients description of the event. It is important to also communicate to the victim that he or she is safe and that it is not his or her fault. Nonjudgmental listening provides an avenue for client catharsis needed in order to begin the process of healing. KEY: Cognitive Level: Application | Integrated Processes: Nursing Process: Implementation | Client Need: Psychosocial Integrity

Which finding would be most likely in a child diagnosed with separation anxiety disorder? A. The child has a history of antisocial behaviors. B. The child's mother is diagnosed with an anxiety disorder. C. The child previously had an extroverted temperament. D. The child's mother and father have an inconsistent parenting style.

B. The child's mother is diagnosed with an anxiety disorder. The nurse should expect to find a mother diagnosed with an anxiety disorder when assessing a child diagnosed with separation anxiety. Some parents instill anxiety in their children by being overprotective or by exaggerating dangers. Research studies speculate that there is a hereditary influence in the development of separation anxiety disorder. KEY: Cognitive Level: Application | Integrated Processes: Nursing Process: Assessment | Client Need: Psychosocial Integrity

A client is diagnosed with terminal cancer. Which situation would the nurse assess as reflecting Kbler-Rosss grief stage of anger? A. The client registers for an iron-man marathon to be held in 9 months. B. The client is a devoted Catholic but refuses to attend church and states that his faith has failed him. C. The client promises God to give up smoking if allowed to live long enough to witness a grandchilds birth. D. The client gathers family in order to plan a funeral and make last wishes known.

B. The client is a devoted Catholic but refuses to attend church and states that his faith has failed him.

Which is the most accurate description of the nursing diagnosis of spiritual distress? A. The client reports no church affiliations. B. The client struggles to identify meaning and purpose in life. C. The client reports seeing the spirit of his deceased wife. D. The client reports that meditation helps him feel connected spiritually.

B. The client struggles to identify meaning and purpose in life.

A nurse is caring for an Irish client who has recently lost a spouse. The client states to the nurse, Im planning an elaborate wake and funeral. According to George Engel, what purpose would these rituals serve? A. To delay the recovery process initiated by the loss of the clients spouse B. To facilitate the acceptance of the loss of the clients spouse C. To avoid dealing with grief associated with the loss of the clients spouse D. To eliminate emotional pain related to the loss of the clients spouse

B. To facilitate the acceptance of the loss of the clients spouse

A client with a history of cerebrovascular accident (CVA) is brought to an emergency department experiencing memory problems, confusion, and disorientation. On the basis of this client's assessment data, which diagnosis would the nurse expect the physician to assign? A. Medication-induced delirium B. Vascular neurocognitive disorder C. Altered thought processes D. Alzheimer's disease

B. Vascular neurocognitive disorder The nurse should expect that this client would be diagnosed with vascular neurocognitive disorder (NCD), which is due to significant cerebrovascular disease. Vascular NCD often has an abrupt onset. This disease often occurs in a fluctuating pattern of progression. KEY: Cognitive Level: Application | Integrated Processes: Nursing Process: Assessment | Client Need: Physiological Integrity

Which nursing intervention related to self-care would be most appropriate for a teenager diagnosed with moderate intellectual disability? A. Meeting all of the clients self-care needs to avoid injury B. Providing simple directions and praising clients independent self-care efforts C. Avoiding interference with the clients self-care efforts in order to promote autonomy D. Encouraging family to meet the clients self-care needs to promote bonding

B. providing simple directions and praising clients independent self-care efforts

A mother questions the decreased effectiveness of methylphenidate (Ritalin), prescribed for her child's attention deficit-hyperactivity disorder (ADHD). Which nursing reply best addresses the mother's concern? A. "The physician will probably switch from Ritalin to a central nervous system stimulant." B. "The physician may prescribe an antihistamine with the Ritalin to improve effectiveness." C. "Your child has probably developed a tolerance to Ritalin and may need a higher dosage." D. "Your child has developed sensitivity to Ritalin and may be exhibiting an allergy."

C. "Your child has probably developed a tolerance to Ritalin and may need a higher dosage." The nurse should explain to the mother that the child has probably developed a tolerance to Ritalin and may need a higher dosage. Methylphenidate (Ritalin) is a central nervous system stimulant in which tolerance can develop rapidly. Physical and psychological dependence can also occur. KEY: Cognitive Level: Application | Integrated Processes: Nursing Process: Implementation | Client Need: Physiological Integrity: Pharmacological and Parenteral Therapies

9. A nursing student asks an emergency department nurse, Why does a rapist use a weapon during the act of rape? Which nursing reply is most accurate? A. A weapon is used to increase the victimizers security. B. A weapon is used to inflict physical harm. C. A weapon is used to terrorize and subdue the victim. D. A weapon is used to mirror learned family behavior patterns.

C. A weapon is used to terrorize and subdue the victim. The nurse should explain that a rapist uses weapons to terrorize and subdue the victim. Rape is the expression of power and dominance by means of sexual violence. Rape can occur over a broad spectrum of experience, from violent attack to insistence on sexual intercourse by an acquaintance or spouse. KEY: Cognitive Level: Application | Integrated Processes: Nursing Process: Implementation | Client Need: Psychosocial Integrity

Which symptom should a nurse identify that would differentiate clients diagnosed with neurocognitive disorders from clients with pseudodementia (depression)? A. Altered sleep B. Impaired attention and concentration C. Altered task performance D. Impaired psychomotor activity

C. Altered task performance The nurse should identify that attention and concentration are impaired in neurocognitive disorder and not in pseudodementia (depression). KEY: Cognitive Level: Analysis | Integrated Processes: Nursing Process: Assessment | Client Need: Physiological Integrity

A client diagnosed with a neurocognitive disorder is exhibiting behavioral problems on a daily basis. At change of shift, the client's behavior escalates from pacing to screaming and flailing. Initially, which action should a nurse implement in this situation? A. Consult the psychologist regarding behavior-modification techniques. B. Medicate the client with prn antianxiety medications. C. Assess environmental triggers and potential unmet needs. D. Anticipate the behavior and restrain when pacing begins.

C. Assess environmental triggers and potential unmet needs. The initial nursing action is to assess environmental triggers and potential unmet needs. Due to the cognitive decline experienced in a client diagnosed with neurocognitive disorder, communication skills may be limited. The client may become disoriented and frustrated. KEY: Cognitive Level: Application | Integrated Processes: Nursing Process: Implementation | Client Need: Psychosocial Integrity

At what time during a 24-hour period should a nurse expect clients with Alzheimer's disease to exhibit more pronounced symptoms? A. When they first awaken B. In the middle of the night C. At twilight D. After taking medications

C. At twilight The nurse should determine that clients with Alzheimer's disease exhibit more pronounced symptoms at twilight. Sundowning is the term used to describe the worsening of symptoms in the late afternoon and evening. KEY: Cognitive Level: Comprehension| Integrated Processes: Nursing Process: Assessment | Client Need: Physiological Integrity

12. A client who has been raped is crying, pacing, and cursing her attacker in an emergency department. Which behavioral defense should a nurse recognize? A. Controlled response pattern B. Compounded rape reaction C. Expressed response pattern D. Silent rape reaction

C. Expressed response pattern The nurse should recognize that this client is exhibiting an expressed response pattern. In the expressed response pattern, feelings of fear, anger, and anxiety are expressed through crying, sobbing, smiling, restlessness, and tension. In the controlled response pattern, the clients feelings are masked or hidden, and a calm, composed, or subdued affect is seen. KEY: Cognitive Level: Application | Integrated Processes: Nursing Process: Assessment | Client Need: Psychosocial Integrity

A preschool child diagnosed with autism spectrum disorder has been engaging in constant head-banging behavior. Which nursing intervention is appropriate? A. Place client in restraints until the aggression subsides. B. Sedate the client with neuroleptic medications. C. Hold client's head steady and apply a helmet. D. Distract the client with a variety of games and puzzles.

C. Hold client's head steady and apply a helmet. The most appropriate intervention for head banging is to hold the client's head steady and apply a helmet. The helmet is the least restrictive intervention and will serve to protect the client's head from injury. KEY: Cognitive Level: Application | Integrated Processes: Nursing Process: Implementation | Client Need: Safe and Effective Care Environment: Management of Care

A preschool child diagnosed with autism spectrum disorder has been engaging in constant head-banging behavior. Which nursing intervention is appropriate? A. Place client in restraints until the aggression subsides. B. Sedate the client with neuroleptic medications. C. Hold clients head steady and apply a helmet. D. Distract the client with a variety of games and puzzles.

C. Hold clients head steady and apply a helmet

7. A woman comes to an emergency department with a broken nose and multiple bruises after being beaten by her husband. She states, The beatings have been getting worse, and Im afraid that next time he might kill me. Which is the appropriate nursing reply? A. Leopards dont change their spots, and neither will he. B. There are things you can do to prevent him from losing control. C. Lets talk about your options so that you dont have to go home. D. Why dont we call the police so that they can confront your husband with his behavior?

C. Lets talk about your options so that you dont have to go home. The most appropriate reply by the nurse is to talk with the client about options so that the client does not have to return to the abusive environment. It is essential that clients make decisions independently without the nurse being the rescuer. Imposing judgments and giving advice is nontherapeutic. KEY: Cognitive Level: Application | Integrated Processes: Nursing Process: Implementation | Client Need: Psychosocial Integrity

A client is experiencing progressive changes in memory that have interfered with personal, social, and occupational functioning. The client exhibits poor judgment and has a short attention span. A nurse should recognize these as classic signs of which condition? A. Mania B. Delirium C. Neurocognitive disorder D. Parkinsonism

C. Neurocognitive disorder The nurse should recognize that the client is exhibiting signs of neurocognitive disorder (NCD). In NCD, impairment is evident in abstract thinking, judgment, and impulse control. Behavior may be uninhibited and inappropriate. KEY: Cognitive Level: Application | Integrated Processes: Nursing Process: Assessment | Client Need: Physiological Integrity

After 1 week of continuous mental confusion, an elderly African American client is admitted with a preliminary diagnosis of major neurocognitive disorder due to Alzheimer's disease. What should cause the nurse to question this diagnosis? A. Neurocognitive disorder does not typically occur in African American clients. B. The symptoms presented are more indicative of Parkinsonism. C. Neurocognitive disorder does not develop suddenly. D. There has been no T3 or T4 level evaluation ordered.

C. Neurocognitive disorder does not develop suddenly. The nurse should know that neurocognitive disorder (NCD) does not develop suddenly and should question this diagnosis. The onset of NCD symptoms is slow and insidious and is unrelated to race, culture, or creed. The disease is generally progressive and debilitating. KEY: Cognitive Level: Analysis | Integrated Processes: Nursing Process: Assessment | Client Need: Physiological Integrity

10. When questioned about bruises, a woman states, It was an accident. My husband just had a bad day at work. Hes being so gentle now and even brought me flowers. Hes going to get a new job, so it wont happen again. This client is in which phase of the cycle of battering? A. Phase I: The tension-building phase B. Phase II: The acute battering incident phase C. Phase III: The honeymoon phase D. Phase IV: The resolution and reorganization phase

C. Phase III: The honeymoon phase The client is in the honeymoon phase of the cycle of battering. In this phase, the batterer becomes extremely loving, kind, and contrite. Promises are often made that the abuse will not happen again. KEY: Cognitive Level: Application | Integrated Processes: Nursing Process: Assessment | Client Need: Psychosocial Integrity

A client is in the late stage of Alzheimer's disease. To address the client's symptoms, which nursing intervention should take priority? A. Improve cognitive status by encouraging involvement in social activities. B. Decrease social isolation by providing group therapies. C. Promote dignity by providing comfort, safety, and self-care measures. D. Facilitate communication by providing assistive devices.

C. Promote dignity by providing comfort, safety, and self-care measures. KEY: Cognitive Level: Analysis | Integrated Processes: Nursing Process: Implementation | Client Need: Physiological Integrity: Basic Care and Comfort

A nurse is making a home visit to a client who is in the late stage of Alzheimer's disease. The client's partner, who is the primary caregiver, wishes to discuss concerns about the client's nutrition and the stress of providing care. Which of the following actions should the nurse take? A. Verify that a current power of attorney document is on file. B. Instruct the client's partner to offer finger foods to increase oral intake. C. Provide information on resources for respite care. D. Schedule the client for placement of an enteral feeding tube.

C. Provide information on resources for respite care

Which should be the priority nursing intervention when caring for a child diagnosed with conduct disorder? A. Modify the environment to decrease stimulation and provide opportunities for quiet reflection. B. Convey unconditional acceptance and positive regard. C. Recognize escalating aggressive behaviors and intervene before violence occurs. D. Provide immediate positive feedback for appropriate behaviors.

C. Recognize escalating aggressive behaviors and intervene before violence occurs. The priority nursing intervention when caring for a child diagnosed with conduct disorder should be to recognize escalating aggressive behaviors and to intervene before violence occurs. This intervention serves to keep the client and others safe. This is the priority nursing concern. KEY: Cognitive Level: Analysis | Integrated Processes: Nursing Process: Implementation | Client Need: Safe and Effective Care Environment

A client diagnosed with neurocognitive disorder due to Alzheimer's disease is disoriented and ataxic, and he wanders. Which is the priority nursing diagnosis? A. Disturbed thought processes B. Self-care deficit C. Risk for injury D. Altered health-care maintenance

C. Risk for injury The priority nursing diagnosis for this client is risk for injury. Both ataxia (muscular incoordination) and purposeless wandering place the client at an increased risk for injury. KEY: Cognitive Level: Analysis | Integrated Processes: Nursing Process: Analysis | Client Need: Physiological Integrity: Reduction of Risk Potential

A nurse assesses a woman whose husband died 13 months ago. She isolates herself, screams at her deceased spouse, and is increasingly restless and aimless. According to Bowlby, this widow is in which stage of the grieving process? A. Stage I: Numbness or protest B. Stage II: Disequilibrium C. Stage III: Disorganization and despair D. Stage IV: Reorganization

C. Stage III: Disorganization and despair

5. In the emergency department, a raped client appears calm and exhibits a blunt affect. The client answers a nurses questions in a monotone using single words. How should the nurse interpret this clients responses? A. The client may be lying about the incident. B. The client may be experiencing a silent rape reaction. C. The client may be demonstrating a controlled response pattern. D. The client may be having a compounded rape reaction.

C. The client may be demonstrating a controlled response pattern. This client is most likely demonstrating a controlled response pattern. In a controlled response pattern, the clients feelings are masked or hidden, and a calm, composed, or subdued affect is seen. In the expressed response pattern, feelings of fear, anger, and anxiety are expressed through crying, sobbing, smiling, restlessness, and tension. KEY: Cognitive Level: Application | Integrated Processes: Nursing Process: Assessment | Client Need: Psychosocial Integrity

The nurse assesses a client as experiencing maladaptive grieving. Which of the following factors confirms the nurses assessment? A. The clients spouse died 12 months ago. B. The client still cries when recalling memories of the deceased. C. The client reports feelings of worthlessness. D. The client reports intermittent anxiety.

C. The client reports feelings of worthlessness.

In planning care for a child diagnosed with autistic spectrum disorder, which would be a realistic client outcome? A. The client will communicate all needs verbally by discharge. B. The client will participate with peers in a team sport by day 4. C. The client will establish trust with at least one caregiver by day 5. D. The client will perform most self-care tasks independently.

C. The client will establish trust with at least one caregiver by day 5

In planning care for a child diagnosed with autistic spectrum disorder, which would be a realistic client outcome? A. The client will communicate all needs verbally by discharge. B. The client will participate with peers in a team sport by day 4. C. The client will establish trust with at least one caregiver by day 5. D. The client will perform most self-care tasks independently.

C. The client will establish trust with at least one caregiver by day 5. The most realistic client outcome for a child diagnosed with autism spectrum disorder is for the client to establish trust with at least one caregiver. Trust should be evidenced by facial responsiveness and eye contact. This outcome relates to the nursing diagnosis impaired social interaction. KEY: Cognitive Level: Application | Integrated Processes: Nursing Process: Planning | Client Need: Psychosocial Integrity

A child diagnosed with attention deficit-hyperactivity disorder (ADHD) is having difficulty completing homework assignments. What information should the nurse include when teaching the parents about task performance improvement? A. The parents should isolate the child when completing homework to improve focus. B. The parents should withhold privileges if homework is not completed within a 2-hour period. C. The parents should divide the homework task into smaller steps and provide an activity break. D. The parents should administer an extra dose of methylphenidate (Ritalin) prior to homework.

C. The parents should divide the homework task into smaller steps and provide an activity break

A child diagnosed with attention deficit-hyperactivity disorder (ADHD) is having difficulty completing homework assignments. What information should the nurse include when teaching the parents about task performance improvement? A. The parents should isolate the child when completing homework to improve focus. B. The parents should withhold privileges if homework is not completed within a 2-hour period. C. The parents should divide the homework task into smaller steps and provide an activity break. D. The parents should administer an extra dose of methylphenidate (Ritalin) prior to homework

C. The parents should divide the homework task into smaller steps and provide an activity break. By dividing the homework task into smaller steps, the child can remain more focused within a limited about of time. Physical activity can release pent-up energy that would distract from task completion. KEY: Cognitive Level: Application | Integrated Processes: Nursing Process: Implementation | Client Need: Psychosocial Integrity

A mother questions the decreased effectiveness of methylphenidate (Ritalin), prescribed for her childs attention deficit-hyperactivity disorder (ADHD). Which nursing reply best addresses the mothers concern? A. The physician will probably switch from Ritalin to a central nervous system stimulant. B. The physician may prescribe an antihistamine with the Ritalin to improve effectiveness. C. Your child has probably developed a tolerance to Ritalin and may need a higher dosage. D. Your child has developed sensitivity to Ritalin and may be exhibiting an allergy.

C. your child has probably developed a tolerance to Ritalin and may need a higher dosage

A nurse in a long-term care facility is caring for a client who has major neurocognitive disorder and attempts to wander out of the building The client states, " I have to get home." Which of the following statements should the nurse make? A. "You have forgotten that this is your home." B. "You cannot go outside without a staff member." C. "Why would you want to leave? Aren't you happy with your care?" D. "I am your nurse. Let's walk together to your room."

D. "I am your nurse. Let's walk together to your room."

A nursing instructor is teaching about the developmental characteristics of clients diagnosed with moderate intellectual disability (ID). Which student statement indicates that further instruction is needed? A. "These clients can work in a sheltered workshop setting." B. "These clients can perform some personal care activities." C. "These clients may have difficulties relating to peers." D. "These clients can successfully complete elementary school."

D. "These clients can successfully complete elementary school." The nursing student needs further instruction about moderate mental retardation because individuals diagnosed with moderate ID are capable of academic skill up to only a second-grade level. Moderate ID reflects an IQ range of 35 to 49. KEY: Cognitive Level: Application | Integrated Processes: Nursing Process: Evaluation | Client Need: Health Promotion and Maintenance

A teenager has recently lost a parent. Which grieving behavior should a school nurse expect when assessing this client? A. Denial of personal mortality B. Preoccupation with the loss C. Clinging behaviors and personal insecurity D. Acting-out behaviors, exhibited in aggression and defiance

D. Acting-out behaviors, exhibited in aggression and defiance

An 8-year-old client diagnosed with attention deficit-hyperactivity disorder (ADHD) was admitted 5 days ago for management of temper tantrums. What would be a priority nursing intervention during the termination phase of the nurse-client relationship? A. Set a contract with the client to limit acting-out behaviors while hospitalized. B. Teach the importance of taking fluoxetine (Prozac) consistently, even when feeling better. C. Discuss behaviors that are and are not acceptable on the unit. D. Ask the client to demonstrate learned coping skills without direction from the nurse

D. Ask the client to demonstrate learned coping skills without direction from the nurse

An 8-year-old client diagnosed with attention deficit-hyperactivity disorder (ADHD) was admitted 5 days ago for management of temper tantrums. What would be a priority nursing intervention during the termination phase of the nurse-client relationship? A. Set a contract with the client to limit acting-out behaviors while hospitalized. B. Teach the importance of taking fluoxetine (Prozac) consistently, even when feeling better. C. Discuss behaviors that are and are not acceptable on the unit. D. Ask the client to demonstrate learned coping skills without direction from the nurse.

D. Ask the client to demonstrate learned coping skills without direction from the nurse. The priority nursing intervention during the termination phase of the nurse-client relationship should include encouraging the client to demonstrate the coping skills learning during the working phase of the nurse-client relationship. KEY: Cognitive Level: Application | Integrated Processes: Nursing Process: Implementation | Client Need: Psychosocial Integrity

A client diagnosed with neurocognitive disorder due to Alzheimer's disease has impairments of memory and judgment and is incapable of performing activities of daily living. Which nursing intervention should take priority? A. Present evidence of objective reality to improve cognition B. Design a bulletin board to represent the current season C. Label the client's room with name and number D. Assist with bathing and toileting

D. Assist with bathing and toileting The priority nursing intervention for this client is to assist with bathing and toileting. A client who is incapable of performing activities of daily living requires assistance in these areas to ensure health and safety. KEY: Cognitive Level: Analysis | Integrated Processes: Nursing Process: Implementation | Client Need: Physiological Integrity

11. Which teaching should the nurse in an employee assistance program provide to an employee who exhibits symptoms of domestic physical abuse? A. Have ready access to a gun and learn how to use it B. Research lawyers who can aid in divorce proceedings C. File charges of assault and battery D. Have ready access to the number of a safe house for battered women

D. Have ready access to the number of a safe house for battered women The nurse should provide information about safe houses for battered women when working with a client who has symptoms of domestic physical abuse. Many women feel powerless within the abusive relationship and may be staying in the abusive relationship out of fear for their lives. KEY: Cognitive Level: Application | Integrated Processes: Nursing Process: Implementation | Client Need: Safe and Effective Care Environment

A client diagnosed with neurocognitive disorder due to Alzheimer's disease can no longer ambulate, does not recognize family members, and communicates with agitated behaviors and incoherent verbalizations. The nurse recognizes these symptoms as indicative of which stage of the illness? A. Confabulation stage B. Early stage C. Middle stage D. Late stage

D. Late stage The nurse should recognize that this client is in the late stage of Alzheimer's disease. The late stage is characterized by a severe cognitive decline. KEY: Cognitive Level: Application | Integrated Processes: Nursing Process: Assessment | Client Need: Physiological Integrity

An older client has recently moved to a nursing home. The client has trouble concentrating and socially isolates. A physician believes the client would benefit from medication therapy. Which medication should the nurse expect the physician to prescribe? A. Haloperidol (Haldol) B. Donepezil (Aricept) C. Diazepam (Valium) D. Sertraline (Zoloft)

D. Sertraline (Zoloft) The nurse should expect the physician to prescribe sertraline (Zoloft) to improve the client's social functioning and concentration levels. Sertraline (Zoloft) is an SSRI (selective serotonin reuptake inhibitor) antidepressant. Depression is the most common mental illness in older adults and is often misdiagnosed as neurocognitive disorder. KEY: Cognitive Level: Application | Integrated Processes: Nursing Process: Evaluation | Client Need: Physiological Integrity: Pharmacological and Parenteral Therapies

A nursing instructor is teaching about pharmacological treatments for attention deficit-hyperactivity disorder (ADHD). Which information about atomoxetine (Strattera) should be included in the lesson plan? A. Strattera, unlike methylphenidate (Ritalin), is a central nervous system depressant. B. When taking Strattera, a client should eliminate all red food coloring from the diet. C. Strattera will be a life-long intervention for clients diagnosed with this disorder. D. Strattera, unlike methylphenidate (Ritalin), is a selective norepinephrine reuptake inhibitor

D. Strattera, unlike methylphenidate (Ritalin), is a selective norepinephrine reuptake inhibitor

A nursing instructor is teaching about pharmacological treatments for attention deficit-hyperactivity disorder (ADHD). Which information about atomoxetine (Strattera) should be included in the lesson plan? A. Strattera, unlike methylphenidate (Ritalin), is a central nervous system depressant. B. When taking Strattera, a client should eliminate all red food coloring from the diet. C. Strattera will be a life-long intervention for clients diagnosed with this disorder. D. Strattera, unlike methylphenidate (Ritalin), is a selective norepinephrine reuptake inhibitor.

D. Strattera, unlike methylphenidate (Ritalin), is a selective norepinephrine reuptake inhibitor. Strattera is a selective norepinephrine reuptake inhibitor. Ritalin is classified as a stimulant. The exact mechanism by which these drugs produce a therapeutic effect in ADHD is unknown. KEY: Cognitive Level: Application | Integrated Processes: Teaching/Learning | Client Need: Physiological Integrity: Pharmacological and Parenteral Therapies

Which developmental characteristic should a nurse identify as typical of a client diagnosed with severe intellectual disability? A. The client can perform some self-care activities independently. B. The client has advanced speech development. C. Other than possible coordination problems, the clients psychomotor skills are not affected. D. The client communicates wants and needs by acting out behaviors.

D. The client communicate wants and needs by acting out behaviors

Which developmental characteristic should a nurse identify as typical of a client diagnosed with severe intellectual disability? A. The client can perform some self-care activities independently. B. The client has advanced speech development. C. Other than possible coordination problems, the client's psychomotor skills are not affected. D. The client communicates wants and needs by "acting out" behaviors.

D. The client communicates wants and needs by "acting out" behaviors. The nurse should identify that a client diagnosed with severe intellectual disability may communicate wants and needs by "acting out" behaviors. Severe intellectual disability indicates an IQ between 20 and 34. Individuals diagnosed with severe intellectual disability require complete supervision and have minimal verbal skills and poor psychomotor development. KEY: Cognitive Level: Application | Integrated Processes: Nursing Process: Assessment | Client Need: Safe and Effective Care Environment

A client diagnosed with vascular dementia is discharged to home under the care of his wife. Which information should cause the nurse to question the client's safety? A. His wife works from home in telecommunication. B. The client has worked the night shift his entire career. C. His wife has minimal family support. D. The client smokes one pack of cigarettes per day.

D. The client smokes one pack of cigarettes per day. Forgetfulness is an early symptom of dementia that would alert the nurse to question the client's safety at home if the client smokes cigarettes. Vascular dementia is a clinical syndrome of dementia due to significant cerebrovascular disease. The cause of vascular dementia is related to an interruption of blood flow to the brain. High blood pressure and hypertension are significant factors in the etiology. KEY: Cognitive Level: Analysis | Integrated Processes: Nursing Process: Assessment | Client Need: Safe and Effective Care Environment

6. A client who is in a severely abusive relationship is admitted to a psychiatric inpatient unit. The client fears for her life. A staff nurse asks, Why doesnt she just leave him? Which is the nursing supervisors most appropriate reply? A. These clients dont know life any other way, and change is not an option until they have improved insight. B. These clients have limited skills and few vocational abilities to be able to make it on their own. C. These clients often have a lack of financial independence to support themselves and their children, and most have religious beliefs prohibiting divorce and separation. D. These clients are paralyzed into inaction by a combination of physical threats and a sense of powerlessness.

D. These clients are paralyzed into inaction by a combination of physical threats and a sense of powerlessness. The nursing supervisor is accurate when stating that clients in severely abusive relationships are paralyzed into inaction by a combination of physical threats and a sense of powerlessness. Women often choose to stay with an abusive partner for some of the following reasons: for the children, financial reasons, fear of retaliation, lack of a support network, religious reasons, and/or hopelessness. KEY: Cognitive Level: Application | Integrated Processes: Nursing Process: Implementation | Client Need: Psychosocial Integrity

Mr. B., age 79, is admitted to the psychiatric unit for depression. He has lost weight and has become socially isolated. His wife died 5 years ago and his son tells the nurse, "he did very well when mom died. He didn't even cry." Which would be the priority nursing diagnosis for Mr. B? a. Complicated grieving b. imbalanced nutrition: less than body's requirements c. social isolation d. risk for injury

a. Complicated grieving

In group exercise, Mr. B, a 79 year old man with major depressive disorder, becomes tired and short of breath very quickly. This is most likely due to: a. age-related changes in the cardiovascular system b. a sedentary lifestyle c. the effects of pathological depression d. medication the physician has prescribed for depression

a. age-related changes in the cardiovascular system

Which of the following nursing diagnoses would be considered the priority in planning care for the child with severe autism spectrum disorder? a. Risk for self-mutilation evidenced by banging head against wall b. Impaired social interaction evidenced by unresponsiveness to people c. Impaired verbal communication evidenced by absence of verbal expression d. Disturbed personal identity evidenced by inability to differentiate self from others

a. risk for self-mutilation evidenced by banging head against wall

A battered woman presents to the ED with multiple cuts and abrasions. Her right eye is swollen shut. She says that her husband did this to her. The priority nursing intervention is a. tending to the immediate care of her wounds. b. providing her with information about a safe place to stay. c. administering the prn tranquilizer ordered by the physician. d. explaining how she may go about bringing charges against her husband.

a. tending to the immediate care of her wounds.

According to the literature, which of the following is most important for individuals to maintain a healthy, adaptive old age? a. to remain socially interactive b. to disengaged slowly in preparation of the last stage of life c. to move in with family d. to maintain total independence and accept no help from anyone

a. to remain socially interactive

Which of the following groups is most commonly used for drug management of the child with attention-deficit/hyperactivity disorder? a. CNS depressants (e.g., diazepam [Valium]) b. CNS stimulants (e.g., methylphenidate [Ritalin]) c. Anticonvulsants (e.g., phenytoin [Dilantin]) d. Major tranquilizers (e.g., haloperidol [Haldol])

b. CNS stimulants (e.g., methylphenidate [Ritalin])

A young woman who has just undergone a sexual assault is brought into the ED by a friend. The priority nursing intervention would be to a. help her to bathe and clean herself up. b. provide physical and emotional support during evidence collection. c. provide her with a written list of community resources for survivors of rape. d. discuss the importance of a follow-up visit to evaluate for sexually transmitted diseases

b. provide physical and emotional support during evidence collection.

Jana, age 5, is sent to the school nurse's office with an upset stomach. She has vomited and soiled her blouse. When the nurse removes her blouse, she notices that Jana has numerous bruises on her arms and torso, in various stages of healing. She also notices some small scars. Jana's abdomen protrudes on her small, thin frame. From the objective physical assessment, the nurse suspects that a. Jana is experiencing physical and sexual abuse. b. Jana is experiencing physical abuse and neglect. c. Jana is experiencing emotional neglect. d. Jana is experiencing sexual and emotional abuse.

b. Jana is experiencing physical abuse and neglect.

In an effort to help the child with mild to moderate intellectual developmental disorder develop satisfying relationships with others, which of the following nursing interventions is most appropriate? a. Interpret the child's behavior for others. b. Set limits on behavior that is socially inappropriate. c. Allow the child to behave spontaneously, for he or she has no concept of right or wrong. d. This child is not capable of forming social relationships.

b. set limits on behavior that is socially inappropriate

Why is it important for the nurse to check the temperature of the water before an elderly individuals gets into the shower? a. The client may catch a cold if the water temperature is too low b. the client may burn himself because of a higher pain threshold c. Elderly clients have difficulty discriminating between hot and cold d. The water must be exactly 98.6 F

b. the client may burn himself because of a higher pain threshold

Which of the following activities would be most appropriate for the child with attention-deficit/hyperactivity disorder? a. Monopoly b. Volleyball c. Pool d. Checkers

b. volleyball

Stanley, age 72, is admitted to the hospital for depression. His son reports that he has periods of confusion and forgetfulness. In her admission assessment, the nurse notices an open sore on Stanley's arm. When she questions him about it he says, "I scraped it on the fence 2 weeks ago. It's smaller than it was." How might the nurse analyze these data? a. Consider that Stanley may have been attempting self-harm b. The delay in healing may indicate that Stanley has developed skin cancer c. A diminished inflammatory response in the elderly increases healing time d. Age-related skin changes and distribution of adipose tissue delay healing in the elderly

c. A diminished inflammatory response in the elderly increases healing time

A client who is in Stage 4 of Alzheimer's disease (AD) is undergoing psychotherapy. Which statement of the client's caregiver indicates effective treatment? a) "The client is less impulsive and emotionally stable." b) "The client has reduced tremors and stiffness in her hands." c) "The client has improved cognition and memory." d) "The client is able to understand and accept problems."

d) "The client is able to understand and accept problems."

Mr. B, age 79, is admitted to the psych unit for depression. He has lost weight and has become socially isolated. His wife died 5 years ago and he lives alone. A suicide assessment is conducted. Why is Mr. B at high risk for suicide? a. all depressed people are at high risk for suicide b. mr. b is in the age group in which the highest percentage of suicides occur c. Mr. B is a white man, recently bereaved, living alone d. His son reports that Mr. B owns a gun

c. Mr. B is a white man, recently bereaved, living alone

The child with autism spectrum disorder has difficulty with trust. With this in mind, which of the following nursing actions would be most appropriate? a. Encourage all staff to hold the child as often as possible, conveying trust through touch. b. Assign a different staff member each day so the child will learn that everyone can be trusted. c. Assign the same staff person as often as possible to promote feelings of security and trust. d. Avoid eye contact because it is extremely uncomfortable for the child and may even discourage trust.

c. assign the same staff person as often as possible to promote feelings of security and trust

An elderly client says to the nurse, "I don't want to go to that crafts class. I'm too old to learn anything." Based on knowledge of the aging process, which of the following is a true statement? a. memory functioning in the elderly most likely reflects loss of long-term memories of remote events b. intellectual functioning declines with advancing age c. learning ability remains intact, but time required for learning increases with age d. cognitive functioning is rarely affected in aging individuals

c. learning ability remains intact, but time required for learning increases with age

A woman who has a long history of being battered by her husband is staying at the woman's shelter. She has received emotional support from staff and peers and has been made aware of the alternatives open to her. Nevertheless, she decides to return to her home and marriage. The best response by the nurse to the woman's decision is, a. "I just can't believe you have decided to go back to that horrible man." b. "I'm just afraid he will kill you or the children when you go back." c. "What makes you think things have changed with him?" d. "I hope you have made the right decision. Call this number if you need help."

d. "I hope you have made the right decision. Call this number if you need help."

Mr. B, age 79, is admitted to the psych unit for depression. He has lost weight and has become socially isolated. His wife died 5 years ago and his son tells the nurse, "he did very well when Mom died. He didn't even cry." Which would be the priority nursing intervention for Mr. B? a. take blood pressure once each shift b. ensure that Mr. B attends group activities c. Encourage Mr. B to eat all of the food on his food tray d. Encourage Mr. B to talk about his wife's death

d. Encourage Mr. B to talk about his wife's death


Kaugnay na mga set ng pag-aaral

Vocabulary Classical Roots Book B Lesson 10

View Set